8
$\begingroup$

Let $m$ be a positive integer and let $f_m(r)=2^{-r}\sum_{i=0}^r\binom{m}{i}$. Clearly $f_m(0)=f_m(m)=1$ and $f_{2r+1}(r)=2^{2r}$.

Conjecture: If $m>12$, then the maximum value of $f_m(r)$ for $r\in\{0,1,2,\dots,m\}$ occurs when $r=\lfloor m/3\rfloor +1$.

Such a weighted sum arises in Coding Theory and in Information Theory, so experts may know the answer to this conjecture. (Computation shows that it is true for $m\leqslant1000$.) The following seem helpful and relevant: (1) the entropy function $H(p)=-p\log p-(1-p)\log(1-p)$, (2) Stirling's approximations, and (3) the MO questions Lower bound for sum of binomial coefficients?, and Estimating a partial sum of weighted binomial coefficients. Alas, I couldn't see how to prove such a precise conjecture. Proving that the maximum is "near" $m/3$, in some sense, may be more tractable.

$\endgroup$
1
  • 3
    $\begingroup$ Astonishing. Computation even shows the assertion true for $m \leq 10000$. $\endgroup$ Apr 11, 2021 at 22:12

2 Answers 2

4
$\begingroup$

I'll give a crude calculation on the back of this envelope.

Assume that $c'm<r<cm$ for some $0<c'<c<\frac12$.

When $i=o(r^{1/2})$, we have $$\binom{m}{r-i}= (1+o(1))\, \left(\frac{r}{m-r}\right)^i\binom mr ,$$ so $$f_m(r)= (1+o(1))\, 2^{-r}\frac{m-r}{m-2r}\binom mr.$$ (The condition $c<1/2$ keeps the ratio below 1 so the sum converges quickly, while the condition $c'>0$ means that $i=o(r^{1/2})$ is enough terms to get most of the sum.)

Consequently, $$\frac{f_m(r-1)}{f_m(r)} = (1+o(1)) \frac{2r}{m-r},$$ which is increasing in $r$ and equals 1 when $r=\bigl(\frac13 +o(1)\bigr) m$.

To get more precision, find a first approximation to the actual error terms. The calculations become more difficult of course.

ADDED. Now I found a bigger envelope and can explain how to get the full result at least for sufficiently large $m$. Having proved that the maximum is close to $m/3$, we will home in on it more accurately. Put $r=m/3+k$ where $k$ is small and $r$ is an integer.

I'll leave out lots of details (often because I don't have them).

First, $$ \binom{m}{r-i} = 2^{-i}\binom{m}{r}\biggl(1 - \frac{3i(3i-6k-1)}{4m} + \cdots\biggr),$$ where I think that is enough terms but I'm not sure (the next term is $O((i^4+i^2k^2)/m^2)$). Summing over $i\ge 0$, $$ f_m(r) = 2^{-r}\binom{m}{r}\biggl(2 + \frac{9k-12}{m} + O((1+k^2)/m^2)\biggr).$$

Dividing two values, $$ \frac{f_m(m/3+k+1)}{f_m(m/3+k)} = 1 - \frac{3(3k-1)}{2m} + O((1+k^2)/m^2). $$

Now consider the class of $m$ modulo 3. For $m=3M$, integer $M$, $$ \frac{f_m(M+k+1)}{f_m(M+k)}=\begin{cases} 1+\frac{1}{2M}+O(M^{-2}), & \text{ for $k=0$};\\ 1-\frac{1}{M}+O(M^{-2}), & \text{ for $k=1$}, \end{cases} $$ so the maximum occurs for $r=m/3+1$ provided the $O(M^{-2})$ is small enough to not interfere. The same phenomenon occurs if $m$ is 1 or 2 modulo 3 (use $k=-1/3,2/3$ for 1, $k=-2/3,1/3$ for 2). There is always a gap of $\Omega(m^{-1})$ that will dominate the $O(m^{-2})$ extras if $m$ is large enough.

To turn this into a theorem, make explicit bounds on the error terms. It could be that easily obtained bounds don't work when $m$ is small, but hopefully that will overlap the computational range.

$\endgroup$
1
$\begingroup$

Unfortunately I cannot recall exactly where I saw it (maybe an exercise in a combinatorics book) but I believe the following is known, the binomial coefficients $$ \left\{\binom{m}{i}\right\}_{0\leq i\leq m} $$ form a super-increasing sequence (last term is bigger than the sum of all previous terms) until $m/3$ (floor or ceiling).

Could this be used to prove what you want?

$\endgroup$
0

Your Answer

By clicking “Post Your Answer”, you agree to our terms of service and acknowledge you have read our privacy policy.

Not the answer you're looking for? Browse other questions tagged or ask your own question.